Sr Examen

Otras calculadoras

  • ¿Cómo usar?

  • Gráfico de la función y =:
  • (x+5)^2-9 (x+5)^2-9
  • x^(7/2)-3 x^(7/2)-3
  • x^3/ x^3/
  • x^4-3*x^2+4 x^4-3*x^2+4
  • Expresiones idénticas

  • y=-(uno / tres)*x^(tres)+9x- seis
  • y es igual a menos (1 dividir por 3) multiplicar por x en el grado (3) más 9x menos 6
  • y es igual a menos (uno dividir por tres) multiplicar por x en el grado (tres) más 9x menos seis
  • y=-(1/3)*x(3)+9x-6
  • y=-1/3*x3+9x-6
  • y=-(1/3)x^(3)+9x-6
  • y=-(1/3)x(3)+9x-6
  • y=-1/3x3+9x-6
  • y=-1/3x^3+9x-6
  • y=-(1 dividir por 3)*x^(3)+9x-6
  • Expresiones semejantes

  • y=-(1/3)*x^(3)+9x+6
  • y=-(1/3)*x^(3)-9x-6
  • y=+(1/3)*x^(3)+9x-6

Gráfico de la función y = y=-(1/3)*x^(3)+9x-6

v

Gráfico:

interior superior

Puntos de intersección:

mostrar?

Definida a trozos:

Solución

Ha introducido [src]
          3          
         x           
f(x) = - -- + 9*x - 6
         3           
$$f{\left(x \right)} = \left(- \frac{x^{3}}{3} + 9 x\right) - 6$$
f = -x^3/3 + 9*x - 6
Gráfico de la función
Puntos de cruce con el eje de coordenadas X
El gráfico de la función cruce el eje X con f = 0
o sea hay que resolver la ecuación:
$$\left(- \frac{x^{3}}{3} + 9 x\right) - 6 = 0$$
Resolvermos esta ecuación
Puntos de cruce con el eje X:

Solución analítica
$$x_{1} = - \frac{\sqrt[3]{243 + 486 \sqrt{2} i}}{3} - \frac{27}{\sqrt[3]{243 + 486 \sqrt{2} i}}$$
Solución numérica
$$x_{1} = 4.8237386487445$$
$$x_{2} = -5.50195979011212$$
$$x_{3} = 0.678221141367624$$
Puntos de cruce con el eje de coordenadas Y
El gráfico cruce el eje Y cuando x es igual a 0:
sustituimos x = 0 en -x^3/3 + 9*x - 6.
$$-6 + \left(- \frac{0^{3}}{3} + 0 \cdot 9\right)$$
Resultado:
$$f{\left(0 \right)} = -6$$
Punto:
(0, -6)
Extremos de la función
Para hallar los extremos hay que resolver la ecuación
$$\frac{d}{d x} f{\left(x \right)} = 0$$
(la derivada es igual a cero),
y las raíces de esta ecuación serán los extremos de esta función:
$$\frac{d}{d x} f{\left(x \right)} = $$
primera derivada
$$9 - x^{2} = 0$$
Resolvermos esta ecuación
Raíces de esta ecuación
$$x_{1} = -3$$
$$x_{2} = 3$$
Signos de extremos en los puntos:
(-3, -24)

(3, 12)


Intervalos de crecimiento y decrecimiento de la función:
Hallemos los intervalos donde la función crece y decrece y también los puntos mínimos y máximos de la función, para lo cual miramos cómo se comporta la función en los extremos con desviación mínima del extremo:
Puntos mínimos de la función:
$$x_{1} = -3$$
Puntos máximos de la función:
$$x_{1} = 3$$
Decrece en los intervalos
$$\left[-3, 3\right]$$
Crece en los intervalos
$$\left(-\infty, -3\right] \cup \left[3, \infty\right)$$
Puntos de flexiones
Hallemos los puntos de flexiones, para eso hay que resolver la ecuación
$$\frac{d^{2}}{d x^{2}} f{\left(x \right)} = 0$$
(la segunda derivada es igual a cero),
las raíces de la ecuación obtenida serán los puntos de flexión para el gráfico de la función indicado:
$$\frac{d^{2}}{d x^{2}} f{\left(x \right)} = $$
segunda derivada
$$- 2 x = 0$$
Resolvermos esta ecuación
Raíces de esta ecuación
$$x_{1} = 0$$

Intervalos de convexidad y concavidad:
Hallemos los intervales donde la función es convexa o cóncava, para eso veamos cómo se comporta la función en los puntos de flexiones:
Cóncava en los intervalos
$$\left(-\infty, 0\right]$$
Convexa en los intervalos
$$\left[0, \infty\right)$$
Asíntotas horizontales
Hallemos las asíntotas horizontales mediante los límites de esta función con x->+oo y x->-oo
$$\lim_{x \to -\infty}\left(\left(- \frac{x^{3}}{3} + 9 x\right) - 6\right) = \infty$$
Tomamos como el límite
es decir,
no hay asíntota horizontal a la izquierda
$$\lim_{x \to \infty}\left(\left(- \frac{x^{3}}{3} + 9 x\right) - 6\right) = -\infty$$
Tomamos como el límite
es decir,
no hay asíntota horizontal a la derecha
Asíntotas inclinadas
Se puede hallar la asíntota inclinada calculando el límite de la función -x^3/3 + 9*x - 6, dividida por x con x->+oo y x ->-oo
$$\lim_{x \to -\infty}\left(\frac{\left(- \frac{x^{3}}{3} + 9 x\right) - 6}{x}\right) = -\infty$$
Tomamos como el límite
es decir,
no hay asíntota inclinada a la izquierda
$$\lim_{x \to \infty}\left(\frac{\left(- \frac{x^{3}}{3} + 9 x\right) - 6}{x}\right) = -\infty$$
Tomamos como el límite
es decir,
no hay asíntota inclinada a la derecha
Paridad e imparidad de la función
Comprobemos si la función es par o impar mediante las relaciones f = f(-x) и f = -f(-x).
Pues, comprobamos:
$$\left(- \frac{x^{3}}{3} + 9 x\right) - 6 = \frac{x^{3}}{3} - 9 x - 6$$
- No
$$\left(- \frac{x^{3}}{3} + 9 x\right) - 6 = - \frac{x^{3}}{3} + 9 x + 6$$
- No
es decir, función
no es
par ni impar